Sei sulla pagina 1di 62

GMAT Study Strategy Authors: Daveformba, Ursula, GMAT Club, Stephen Bolton, Erin

Revised 1/25/05
I’ve searched all over the internet and read all the major books on preparing for
the GMAT. The main thing that current GMAT prep books do well is to help identif
y what you’ll face on the test. What they don’t do well is to help you to know how t
o actually study each type of question and how to apply the strategies effective
ly. I’ve struggled to try and figure out how take a more active posture in studyin
g such that information is not only retained, but efficiency improved. I’m tired o
f hearing, “just do a lot of problems.” That’s not useful. That was what people were t
elling me. One person, who attained a 740 on the GMAT at 46 years of age, the fi
rst time she took the test, stated how important it was to stay active and not b
e passive while studying. The point of this whole guide is on how to become an a
ctive learner for the GMAT and avoid being passive. (Read her Story here) As I w
as studying, I realized that I had to lay down some very specific strategies for
myself to become more active in my learning process. This was because of a GMAT
practice test I took where I got a 600. I had studied the Quant quite a bit, bu
t not the verbal. For verbal, I went mostly off of what sounded right to me. I n
eeded…. I wanted to come up with a way to approach the rest of my studying more ef
fectively because there seemed to be too much memorization or gut reliance going
on in my test taking skills. I knew that going by what sounds right wasn’t the be
st way approach the real GMAT. This guide was created as a result of brutally as
king myself specifics on how I could and would improve in each area of the GMAT.
How does one move from, “just do a lot of problems” or “study hard” to “studying effectiv
ely” and most importantly…. Getting results? As this is a guide that is being refine
d and added to, I hope you’ll join me in letting me know what you feel needs to be
re-worked or further explained by you or me to improve it. This will be of grea
t benefit to you and me as well as others that might need this in the near futur
e. Over time, I’ve received permission from several authors and teachers that have
provided great strategy guidelines and have inserted them into this document. I’m
not sure how you have discovered this GMAT study guide as I have not advertised
it anywhere. In any case, I say good for you because it shows that you’re doing y
our homework. Hopefully, you weren’t just looking for the easy way out? Trust me,
no matter how much you look, you won’t find a study guide better than this on the
internet. It’s not that I searched myself and judged this guide to be the best. Ra
ther, anyone who’s creating anything that’s even close is trying to sell you somethi
ng. I’m not. There’s no extended version of this guide that I sell. This is just me
helping others. I do only ask for two things as you use this guide. 1) I have in
cluded links to books and study guides where I do receive very small referral fe
es. Therefore, I ask that you use the links in this document to purchase the GMA
T books you’ll be buying used or new to help support this web site you found this
document on. (By the way, I have and always will only suggest resources that hav
e been their weight in gold. I don’t work for or receive kickbacks ever and never
well. As you progress through this document and the associated links, I think yo
u’ll see that I’ve put in a significant amount of time into putting this all togethe
r) 2) Provide me with feedback on how to improve this document such as informati
on you may want to submit to me. For example, I have received some feedback that
they would like to see more specifics on Math prep. As this is a lot of work, I’m
taking my time on that, but have included some information in this latest revis
ion. I have compiled a web page of various resources to file my research and org
anize it for easy browsing. You can find it at Future MBA Resources (FMR) . I ha
ve created one specific page just for GMAT resources. This will be where you’ll fi
nd a wealth of resources and free tests to help you on your journey. For example
, I compiled a spreadsheet that lists the difficulty of every question in the GM
AT Official Guide from GMAC the creator of the test. Dave daveformba@hotmail.com
My Blog Erin http://www.sentencecorrection.com/forums/index.php?act=Mail&CODE=0
0&MID=2 Dave’s Editorial Corner As I’ve received quite a bit of e-mail asking me que
stions about the GMAT and managing difficulties in progress, I’ve decided to add t
his new section that includes my responses.
GMAT Question Answered Dear Dave,
Thank you very much for putting your GMAT strategy guide on the net, it s the be
st so far! I am not a native speaker, I am struggling with my preparation. I fee
l like I am not progressing, can you please advise something on this regard? I r
eally appreicated it. Best Regards, "T"
Answer: That’s a difficult question to answer. I’ll try my best. I’m hoping that you’re
doing well in Math. If this is not the case.. then the answer may be in how you
review and think through why you get the wrong answer and ESPECIALLY why you got
the right answer. You have to do your best to commit to daily reviews of the on
es you got
wrong from the previous day. OG is the best book hands down. Review all the answ
ers in it. Moreover, you should review all the ones you’ve been getting wrong at l
east once a week. If you keep getting them wrong.. then you’re approach has been t
o just solve the problem.. rather than understand the concepts. If you re intere
sted in which questions are difficult, medium or easy, you can check out the spr
eadsheet that I pulled from GMAC to help you. For English.. If you think about i
t… verbal is a math skill too. There are rules and guidelines and many ways to app
roach the answer. You have to understand the basics. I started to do that in my
study guide. You have to make sure you memorize the basics. Then it comes down t
o repetition and practice. Don’t just try to do as many problems as you can when i
t comes to English. Understand not just why you got the question right.. buy exa
ctly why the other choices were poor. Don’t just say.. makes sense or sounds right
. It s better to figure out which rule it broke or which concept the wrong answe
r didn’t adhere too. By the way… take a practice full exam… not just verbal or math… but
a full exam at least 1 per week. Find any test you can get your hands on. It do
esn’t matter if the test is adaptive or not. And put yourself under the same time
pressures of both the test taking time and the length of breaks. This part is cr
ucial. What I described is the big picture approach for the GMAT. When it comes
to discipline… there are two things that I would suggest. 1) Take a class, but qui
t the class if the teacher sucks. They should blow your socks off otherwise, it’s
a waste of your time. 2) Whether you take the class or not… get to a place where y
ou can focus and study with little or no distractions. I study at universities m
yself. For example, while I wrote my Business School application essays and stud
ied for the GMAT, I went to Stanford at their 24 hour study center and took some
drinks and sandwiches everyday. It was not unusual for me to get there by 6pm a
nd leave by 1am. Most Universities like Stanford don’t require a parking permit af
ter 5pm weekdays and no permit is required on the weekends. I called and e-maile
d my friends that I wouldn’t be around much because of my studies and I canceled a
lot of appointments and activities in my life. I gave myself permission once a
week to go see a movie after taking a practice test because my brain was fried a
nyway. I m definitely not an expert at the GMAT, but in my humble opinion... I v
e written what I believe to be a good approach to studying for the GMAT. Not all
ways work for everyone. I hope you find what works the best for you. But trying
various methods such as what I ve outlined will hopefully get you there.
Best wishes
The Hard Facts About Your GMAT Score See how the chart below shows GMAT scores i
n 3 Tiers? I think people generally have a vague notion that 3 tiers exists, but
with no hard evidence like this chart... who s to say otherwise
Here s a follow-up to the last chart. Again 3 tiers of applicants regarding GMAT
scores.
Here s an interesting graph from Kellogg s 2004 entering class. This is a good G
MAT question possibility. If 5282 applied to the 2-year MBA class resulting in 4
69, what percentage of students were admitted with the various GMAT brackets of
scores as shown in the graph? When you do the math (Taking into account that Kel
logg admits 12% more than needed for instances where the applicant chooses anoth
er school), you get the following: 640 or less GMAT = 1056 apps with 42 accepted
at 4% acceptance. 650-690 GMAT = 1584 apps with 158 accepted at 10% acceptance.
700-740 GMAT = 2059 apps with 252 accepted at 12.2% acceptance. 750-780 GMAT =
581 apps with 74 accepted at 12.7% acceptance. That means the 2004 class had 25%
of all students who scored 700 or more accepted. However, the actual student po
pulation is actually 62% people who scored 700 or more. This is actually a great
challenge and encouragement as I thought it was much worse. So 38% of the class
scored 690 or less? That s cool. 700-740 is only 2% more likely to receive acce
ptance than those who had 650-690. Not much of a difference. I think this is why
we ought not to kill ourselves for not getting 700 and for not killing our self
if we do get 700 or more and don t get into a top Business School. That 2% can
t hurt though.
Comments from a GMAT Instructor Last year, I received some feedback from a 12 ye
ar veteran Princeton Review GMAT instructor. Here are some comments he wrote in
an e-mail to me that may be helpful for you. What we do for high scoring student
s to make them get their scores up (in a classroom setting) is to run classes th
at focus on ONE AREA. For instance, Statistics and GMAT Miscellany, SC, CR, Data
Sufficiency. People at or above the 600 level have to revamp their study plan.
Random info dump won t be effective anymore. If you can get a 600, then you basi
cally know most of the information there is on the test. What you are probably l
acking is a systematic way of using that information and an understanding of goo
d GMAT strategy. I told the instructor that I wanted to focus on probability a b
it more. His reply on that was as follows: You are basing some of your plans and
concerns on forums. In the last 5 "Test Recons" there have been an average of 1
.3 probability questions per test. 1.3 ... think of all the energy and time dedi
cated to probability in the forums and think of what kind of wasted effort is po
ured into that subject. Forums are 95% noise, 4% good intention, and 1% informat
ion. If you think of it that way you will actually look at them with an entirely
new eye and perhaps glean the little there is there to be had. Test prep compan
ies sometimes fall for the same sort of
paranoia. They respond to the concerns of their students and when those students
fears are being fed and influenced by the forums a nasty little spiral starts
to form. Ask how many people teaching the GMAT took the GMAT in the last 6 month
s I think you will be shocked and dismayed. While I do not think that someone ha
s to take the test often to be a good teacher, a good teacher will take the test
often to be able to screen the noise. The graphic below is part of how I assess
student needs. Each red zone is a plateau. It is a score range that is REALLY h
ard to break through. You are in the plateau that the course is designed to get
people to. Don t believe the nonsense out there. There is no super effective one
-size-fits-all solution to maximum GMAT improvement. Achieving and moving throug
h each plateau has its own unique set of needs and concerns.
What you’ll find in this document 1) GMAT online forum resources and why they are
helpful. 2) Study habits of 19 people who achieved 700+ GMAT scores. 3) How to t
rack your progress and improve your review process 4) What kind of schedule shou
ld I use? What order should I go through the material? 5) Ongoing help resources
– Daily e-mail questions 6) What GMAT resources should I use? In other words, wha
t books and resources out there are good and what sucks. 7) Should I use a Test
Prep Company? 8) What kind of study schedule should I use? 9) A summary study gu
ide for each section of the GMAT 10) A study guide on probability. There just ar
en’t any books out there that cover probability well. GMAT online forum resources
and why they are helpful A couple of websites I found in my research were www.te
stmagic.com and www.gmatclub.com. Both are tremendous resources because you can
post questions there
and people will help you to understand why one answer is right and another is wr
ong. You should see the generous amount of time people take to post responses. O
ne thing I noticed was how a few people who did well on the actual GMAT mentione
d how the forum benefited them so much. They went on to mention how they stayed
active in the forums. A good forum is created when people take the time to expla
in an answer and not to just simply try to guess at the answer or write out a qu
ick formula to show how you got the answer. How did you get the formula? What le
d you to start out with certain steps? What concepts were used in your solution?
Putting your answer in this kind of context is helpful and most importantly for
ces the person who is posting the answer to explain the answer through fact and
theory. To write out the answer requires a reasonable understanding of the conce
pt being tested. To explain the answer to someone is literally like being a teac
her. Teaching a topic is the best method of learning. Now that’s active learning!
Here’s an example of a typical posting. Posting Hi Guys! Help me with this one. Q.
What is the volume of a certain rectangular solid? (1) Two adjacent faces of th
e solids have areas 15 and 24 respectively. (2) Each of two opposite faces of th
e solid has area 40. I think statement (1) alone is sufficient to answer the que
stion. If not, please explain me why? Thanks XXXXX Reply #1 Statement (1) alone
is not sufficient as we can get multiple results for volume: Face 1 can have an
area of 15 with sides 15 & 1: 15 * 1 and Face 2 can have an area of 24 with side
s 1 * 24. The volume would be 15*1*24 = 360 Face 1 : 5 * 3 Face 2 : 3 * 8 Volume
: 5*3*8 = 120 Statement (2) alone is not sufficient as we have info only about
2 sides. Combining Statement (1) & (2), Areas with 15, 24 & 40 correspond to thr
ee sides with lengths measuring 5,3 & 8. Ans should be C. Please let me know the
correct answer. Reply by Original Poster
The answer was infact C.
Reply #2 According to B each opposite face of solid has an area of 40.So all the
faces of the solid are equal in area.So the solid is a cube.So each edge is (40
)^1/2 long .So we can definitely find volume with B.So "B" should be the correct
answer. Please let me know if I am missing some point over here. Reply #3 State
ment 2 just says that each of two opposite faces of the solid have an area of 40
. As therez no info about the remaining 4 faces of the cube, this statement alon
e is insufficient. Thanks, XXX I found about 19 people who scored 700+ and found
quite a few similarities in their study habits and what books they chose to pre
pare with. Here is a summary of that analysis. Books used (98% used the followin
g)
1. Kaplan GMAT book 2. Princeton Review book 3. Official Guide for GMAT 4. Power
Prep Tests
Book/Resources used (About 4-5 people used the following)
1. Kaplan 800 - Hard GMAT questions - more than a few argued that this had the
hardest questions they found.
2. 800-score tests – A good resource for tests. 3. GMAT Plus – A good resource for t
ests. 4. DeltaCourse - primarily a combination, probability, permutation advance
d study guide
with plenty of sample questions to practice with.
5. Barron s - All said study guide part sucked, but questions were useful. 6. Ve
rbal Workout for GMAT -Princeton
Books/Resources all said sucked.
1. Arco Master the GMAT CAT – Many of the answer are wrong. But the study guides
are good.
2. Petersons
3. Crack-GMAT – Don’t waste your time or money on this one according to many. 4. Kap
lan’s in-person GMAT prep course – a few who did it said that it didn’t help.
Study tips that every single 700+ scorer mentioned. 1. Record all mistakes on wr
ong questions and revisit them no sooner than 5 days. If still getting wrong, th
en zero on the subset s and go for more help.
2. Always read the explanation in the book to see why you get it wrong and take
enough
time to really understand the concept.
3. Pick a date and register. Don t study then register. There was a tendency to
not take
prep time as seriously.
4. Quite a few went through OG questions more than once. Not all questions on th
e
repeat though; Mainly verbal sections and Quant that were areas of weakness.
5. Wrote practice essays at least 20 times under timed conditions. 6. Visit the
test center before the day of test. Many did so a week or two before. 7. Time sp
ent by most averaged between 3 and 4 months.
8. All averaged 2-3 hrs per day and no less than 10-12 hrs on the weekends.
9. Took advantage of every break during the GMAT. Take a bathroom break, some
washed their face to wake up, got fresh air.
10. Study in blocks. One person mentioned studying in 80 minute blocks without g
etting
up to build stamina. Another person mentioned studying in 75 minute blocks with
exact numbers of questions that are on the real GMAT to build stamina and skill.
Many agree that the ability to handle the time constraints is nearly as importa
nt is solving the problem. Scores in the actual GMAT are worse if you leave an a
nswer blank than if you get the answer wrong. Interesting note: The Official Gui
de for GMAT contains questions that are in the 550 to 650 difficulty range. Let’s
Get Started The first step is to get a bit analytical. When going through questi
ons, use either one of the following tracking sheets. http://home.comcast.net/~d
ave.kim/GMATTracker.xls http://home.comcast.net/~dave.kim/GMATAnswersForm.zip
What this will help you do is track your progress in a way that can tell you in
a measurable way whether carelessness or lack of skill caused a wrong answer. In
other words, using this sheet will help you to zero in on the what you can targ
et for improvement. Instead of saying, I need to improve in math you’ll be able to
say, “I need more work in probability, more work in stamina or more work in seein
g various forms of misplaced modifiers in sentence correction questions. One thi
ng is for sure, most people find that 50% or more of the reason why they get a q
uestion wrong is due to carelessness, or a loss/lack of stamina. Without a good
tracking system, you may not be able to measurably know by how much this is true
or whether it’s something else. It’s also good to use the notes area frequently to
note the kind of subcategories of question types or if it was a concept you didn’t
know or a concept you did know, but weren’t solid on yet. When you start out, it’s
helpful to spend a week or two un-timed. Then as you progress, start putting you
rself under moderate time pressure to at least record how long it’s taking you to
do a particular question. It would be completely ridiculous to record how long i
t took you to answer every question. It’s far better to track yourself in blocks o
f time. For example, with 12 questions, you should have X minutes left and with
24 questions, you should have X minutes left and so on. Another excellent tracki
ng method is to put one, two or three X’s in the slow box if you took longer than
real GMAT time conditions to answer the question. This is one strategy I rely on
heavily. I use both block time tracking and slow progress tracking actually, bu
t find the slow progress tracking the most helpful. Another good strategy is to
go through questions in the following manner. 1) Do about 40 questions at a time
. In this way, you should be giving yourself 80 or so minutes to complete them.
This will help you to get used to doing questions in blocks similar to the GMAT.
This will build your physical stamina and build longer periods of mental toughn
ess and focus. 2) Check your answers and DON’T look at the explanation of the answ
er. Circle the right answer in red and the sheet. 3) Go back, and use the teachi
ng method as explained earlier to resolve the problem. Note whether you find the
reason why you got the answer wrong as careless or due to concept error in the
chart. A concept error is defined as an error where you didn’t understand the theo
ry enough to know which steps and in what order to solve the problem. It could a
lso be a relevant piece of knowledge that you forgot or haven’t learned yet. See t
he sample posting above to get an idea of what a teaching method solution looks
like. 4) Next go through the explanations to try and understand if you your meth
ods were right or if the book offers an alternative solution method. There is mo
re than one way to explain or solve many problems, so don’t feel like you have to
follow the particular explanation in the book. If your method solved it, and you
can repeat it, then stay with what you know as long as it holds up to scrutiny
when slight changes in the
problem don’t allow your method to work. In which case, you’ll need to adapt the boo
k concept or dig deeper into asking yourself whether you really understood the c
oncept to begin with. By the way. It’s really helpful to ask yourself two question
s when you’re done with each test problem re-solve 1) would I be willing to bet $2
0 that I really understood the concept being tested? 2) Did I write out my expla
nation well enough that most anyone could look at it and understand how I solved
the problem? Ongoing Help You can have e-mails of GMAT questions sent to you on
a daily basis. The two popular ones are Delta Course and Manhattan GMAT
• •
For Delta Course, go to http://www.deltacourse.com/gmat/gmat-email.asp For Manha
ttan GMAT, go to http://www.manhattangmat.com/ Select the Student Center menu an
d create a profile. At the bottom of the profile registration, you’ll be asked if
you want the daily question or not. This is free by the way.
What Resources should I use?

Kaplan Workshop – This is the CD you get when you sign up for the KAPLAN class. It’s
awesome. Both the Quantitative and Verbal reviews are helpful. I struggled with
Sentence Correction and the Workshop exercises and walkthrough started from the
basics and had mini-quiz’s to drill you on the material as you went through. The
number properties review was very helpful as well. You can get it at scoretop ht
tp://www.64168.com/bbs/uploads/workshops.rar. It’s a compressed file. If you don’t w
ave winrar to decompress it, you can download it at www.rarlab.com for free.
• • • • • • •
The Official Guide to the GMAT – Can be obtained from Amazon Princeton Review, Cra
cking the GMAT 2005– Can be obtained from Amazon Kaplan GMAT 2005– Can be obtained f
rom Amazon DeltaCourse – Only available online Powerprep Tests – ETS, the company th
at makes the GMAT. GMAT Tutor has a good Math basics reference sheet to help you
memorize formulas and math basics To see if your GMAT essay abilities are up to
snuff, you can use GMAC’s services to review some sample essay’s. They’ll use the exa
ct same software that is used on the GMAT to review and score your essay. You ca
n get it at:
• • •
Test Magic – www.testmagic.com - Excellent forum to get questions answered or expl
ained. GMAT Club – www.gmatclub.com – Similar to Test Magic GMAT Club Prep material
- It’s pretty good.

http://daveformba.blogspot.com – my weblog. Obviously, this is where you got this
guide from, but you’ll find that I make enough updates and additions to make it wo
rth your while to check it regularly.
• • •
GMAT plus material - A great set of GMAT questions for all subject matters on th
e GMAT. Print them out and go through them. Scoretop.com – If you don’t have access
to GMAT software or your CD got fried, go to http://www.scoretop.com/forum/forum
_topics.asp?FID=6 Future MBA Resources – This is my GMAT and MBA resources web sit
e.
Should I use a Test Prep Company? It’s really up to you. If you find that you stru
ggle with discipline or life is really busy. Then you should. Highly recommended
is Princeton Review (www.princetonreview.com) and Veritas (www.veritasprep.com)
If you have the money and desire more personal instruction, check out Manhattan
GMAT (www.manhattangmat.com). They’re very good. What kind of schedule should I u
se? What order should I go through the material? Again, it’s really up to you, but
here is a basic plan that most people have used that showed great results. 1) G
o through the Kaplan Workshop FIRST. It’s the best Basic review that I’ve found of M
ath and Verbal. 2) Go through the Princeton Review Quant and Verbal Study Guide
section 3) Do at least 50 questions of each type. 4) Take the one Princeton Revi
ew Test from the CD. Use it as a reference point to see where your at. Do it und
er the strictest time conditions. 5) Finish the Princeton book
6) Get the Deltacourse material and go through it.
7) At the same time, begin going through the Kaplan questions.
8) At a MINIMUM take one full GMAT practice test a week. Whether it’s Kaplan or
Princeton. Save the last Powerprep for when your about 2-3 weeks away from the t
est. The Powerprep tests are well known to provide results quite similar to the
real GMAT. The Kaplan ones are traditionally harder and therefore give lower ave
rage scores. Don’t worry about the scores if they are lower. If you are worried, g
o to http://www.gmatclub.com/content/resources/estimator/index.php. There, you’ll
find a score estimator. It will take the Kaplan score you get and average it out
to a more realistic score. 9) Go through the Official GMAT Guide. Do all the qu
estions.
10) If you have the time, use the GMAT Plus material for an additional source of
questions to go through.
11) Take time to do a weekly review of the verbal and quant formulas and referen
ce
sheets you made. www.gmattutor.com has the beginnings of an excellent Math Basic
s reference sheet. Bar none, the Princeton Review for Verbal is your best bet. S
ee below for my summarized version of it. 12) Do at least a dozen practice essay
s. Do each one under timed conditions. 13) Don’t study the day before the test. An
d make sure to visit the test center at least a week before the test. Bring ear
plugs to the test center if noise will be an issue. Get used to studying with th
em if you do though. 14) Create regular review sessions where you go over all th
e questions you got wrong on a weekly basis. Make a binder or something similar.
Problem Solving and Data Sufficiency Study Strategy Kaplan and Princeton are go
od for most topics except for Combination, Permutations, Probability and Countin
g. They don’t cover these Quantitative math skills very well. The best resource I’ve
found is from Deltacourse Their material is highly focused on these topics. It
was extremely helpful. For $27, I wouldn’t even hesitate about ordering it online.
You can’t buy it in bookstores. For a scaled down version of this, go to Appendix
A of this Guide. For every Data Sufficiency and Problem Solving question you ge
t wrong, write out in long hand the solution. Don’t just write out the formula. Th
is will help you process the theory being tested. Explain why you did certain st
eps. I call this the teaching method explanation. Here’s a number Property table f
rom Kaplan Workshop that neither the Kaplan or Princeton books provided and was
extremely helpful as I realized that there were number properties I didn’t remembe
r or recall having learned years ago. Number Properties 0 1 2 7 -3 .5 Pi √6 -19/2
51/3 Even negative x Odd negative Odd negative + Odd negative Even Positive – Odd
Negative Odd negative / Even positive Points of emphasis: Integer X X X X X Posi
tive X X X X X X X X X X X X X X X X X X X X X X Negative Even X X X X Odd X X X
Prime
O is an integer, it is neither pos of neg. Negative numbers are not prime 0 and
1 are not prime Reading Comprehension Study Strategy 1) Aggressively read each p
aragraph for its main idea. If you can’t write down in a few words what the point
of each paragraph is, you weren’t reading actively enough. You should jot down the
following. a. Main idea or primary purpose b. Organization/Structure c. Tone or
attitude of author (if applicable)
Note: Be careful to not write facts down. It’ll bog you down and usually results i
n a loss of the big picture and moves you to focus to much into the details. 2)
Note any trigger words, same train of thought words, yin-yang parallelism. See t
he Princeton Review book for more explanation. 3) Weed out possible disputable a
nswers. Vague, wimpy answers are often correct over stronger statements. ETS doe
sn’t want to get many complaints that a particular answer that was strongly stated
, that exceptions could arise. ETS would rather play it safe. 4) Minority passag
es are often positive in tone and answers tend to be positive in tone as well. A
gain, ETS, wouldn’t want to look prejudiced. 5) Always eliminate bad choices first
before answering. You’ll almost always be able to narrow down to 2-3 and that sig
nificantly improves your odds of getting the question right. 6) Read the entire
passage before answering the questions. Other books say skim, but it’s not always
successful with more difficult passages where minute details change meanings of
the passage and could get you going down the wrong path on inference or main poi
nt questions. Give yourself 3 minutes or less. 7) For Inference questions, (Note
: these are usually the hardest of all RC questions) go find the general area be
ing referenced. Read a bit above or below it and then make your choices. Don’t go
by memory. This is going to cause more problems that be helpful with saving time
in the long run. Your answer should never contradict the main point of the pass
age. 8) Most people get main point and inference questions wrong so focus more c
arefully on these. 9) "According to the passage/author" question type of questio
ns. Whenever you see this question, tell yourself, "Stop and stop thinking. I ne
ed to FIND, not think."
10) For main point or central idea type of questions, re-read the first and last
sentences of each paragraph before making elimination choices and answering. Ge
tting the overall structure is really helpful before answering. 11) In Summary:
Consider weeding out answers that a. Are disrespectful to others/professionals.
ETS doesn’t like to be disrespectful. b. Too strong an answer. Use of words like “on
ly”, “definitely”, “positively” c. Condone/approve prejudicial attitudes. ETS doesn’t like
o be disrespectful. 12) For a much more detailed strategy for Reading Comprehens
ion, go to Appendix D Sentence Correction Study Strategy Overall strategy 1) Mem
orize the 8 Princeton Review methods recognize bad sentences. I’ve summarized the
key points below. The Kaplan Workshop, Princeton Review/Kaplan and Arco’s Sentence
Correction study sections are the key resources that I’ve learned the most from.
And believe me… I had a lot to learn. 2) Initially, do 20-30 questions and explain
what error you saw in the sentence or didn’t see. For wrong answer choices, list/
explain specifically what Princeton Review rules this answer choice violated. So
metimes, it’s a combination. Also explain why the sentence was tricky if it’s a diff
icult one. For example try to identify if there were long prepositional phrases
between a verb and subject agreement or a long adjective modifying phrase instea
d of an easy single word modifier. 3) For all future questions, make sure you wr
ite, verbally, or mentally talk through the rule that was violated. Make notes o
n a scratch piece of paper. If you pick an answer and can’t explain EXACTLY what m
ethod the answer corrects, then you are just guessing or going off of what sound
s good--- Therefore, reinforcing a bad habit. Spotting bad sentences is the key
to doing well on sentence structure test questions. 1) Pronoun error a. Plural a
nd single – once you start with one, you need to stay in the same quantity. Singul
ar Pronouns (Memorize these. Period) Hint: Do you see the categories I setup? It’s
SANE to memorize this Some--Any— No— Every-Everyone Everybody Someone Somebody Eith
er Neither One Each
Everything Anyone Anybody Anything
Something No one Nothing Nobody
Whoever Whomever
His
Be aware that group, jury, team, country, family are singular. Society today use
s them sometimes as plural. This is because these act as a single unit when they
do something. Plural Pronouns (Memorize these. Period) Both Few Their Others Ma
ny Several
Singular and Plural Pronouns – depends on whether the noun is singular or plural (
Memorize these. Period) Some More Most All
The plural and singular clause error When two nouns are in the sentence doing an
action together but they are linked with Along with Together with With As well
as In addition to Accompanied by … this does not make the following action they do
plural. Only “and” can take the two singulars and make their action plural. For exa
mple Janie, with her poodle limping behind her, walks to the dog park. Explanati
on: Janie is singular. The poodle is singular. They both do the action together,
but the use of “with” means that we need to keep the verb singular. “Walks” is singular
and “Walk” is plural Remember, a verb that ends with an –s is singular. b. Pronoun re
ference error- referring pronoun is not correctly placed. c. Relative pronouns a
re often used incorrectly today. 1) Referring to things or animals – that, which 2
) Referring to people—who, whom 3) They – be careful that you don’t use this unless yo
u’re positive there is a referring noun. Today we often use “they” to replace the use
of a proper noun which it is not. It’s a Pronoun.
2) Misplaced Modifier (modifiers must stay close to home) a. Definition: Sentenc
es that begin with a verb, adjective+verb, adjective phrases need to be followed
by the noun or pronoun they are modifying. Usually end with –ing. b. Example: Com
ing out of the department store, John’s wallet was stolen. “Coming” is the modifier. W
as john’s wallet coming out of the store? Incorrect sentence. Possible solution to
look for: a) Correct the reference
b) Put a noun or pronoun into the 1st part of the sentence turning the
1st part into an adverbial clause. Thus can stand apart without needing to watch
the modifier. Absolute Phrases: Introduction 1. Introduction Definition and rul
es. An absolute phrase is a modifier (quite often a participle), or a modifier a
nd a few other words, that attaches to a sentence or a noun, with no conjunction
. An absolute phrase cannot contain a finite verb. Absolute phrases usually cons
ist of a noun and a modifier that modifies this noun, NOT another noun in the se
ntence. Absolute phrases are optional in sentences, i.e., they can be removed wi
thout damaging the grammatical integrity of the sentence. Since absolute phrases
are optional in the sentence, they are often set off from the sentence with com
mas or, less often, with dashes. We normally explain absolute phrases by saying
that they modify entire sentences, rather than one word. This is an important co
ncept, since many similar phrases that we work with modify other words. For exam
ple, adjectives modify nouns, and adverbs can modify verbs, adjectives, and othe
r adverbs. That said, however, in some cases, it seems to make more sense to say
that absolute phrases modify nouns. We will look at some of these examples a bi
t later. First, let s look at some examples of absolute phrases: Examples of Abs
olute Phrases: The absolute phrases look like this: • • • Her determination stronger t
han ever, Nexisa resolved not to give up until she had achieved her dreams. The
sun shining bright and the pale blue sky forming a backdrop of the Sacre Coeur,
Carl stepped into his future as a traveler and observer. Still young boys, Matt
and Erin Billy awoke early one Christmas morning with sleepy eyes, completely un
aware that they were sleeping not in the beds they had gone to sleep in, but in
one of their presents that year -- a new set of bunk beds. • We finished the heart
y meal quickly, our appetites satisfied, our minds at peace.

All things being equal, the active voice tends to be correct more often than the
passive on standardized tests.
Please notice that in every case the absolute phrase provides some sort of infor
mation that works to put the whole sentence or idea in context. Please also noti
ce that the absolute phrases themselves do NOT contain verbs, nor are they conne
cted to the main sentence with a conjunction. Finally, please notice that the pr
imary components of most (but not all) of these absolute phrases are a noun + a
modifier, although it is possible to use only a modifier. Here is the next patte
rn we should know:
2. noun + participle This is one of the most common ways to form an absolute phr
ase. It might be helpful for some people to imagine this pattern with a verb bet
ween the noun and the participle. For example, if you say The question was still
unanswered, you have a complete sentence; if, on the other hand, you say The qu
estion unanswered and you then attach that phrase to a main sentence, then you h
ave an absolute phrase. Here are some examples. The absolute phrases look like t
his. • • • The question still unanswered, the teacher decided to address the confusion
of her students more closely. The train running late, we decided to get off at
the next stop and take a taxi home. There are many industries in California vita
l to its economy, with technology being one of the most important. Compare these
sentences with the verbs and conjunctions in them: • • • The question was still unans
wered, and the teacher decided to address the confusion of her students more clo
sely. The train was running late, so we decided to get off at the next stop and
take a taxi home. There are many industries in California vital to its economy,
and technology is one of the most important. Important! Although many of these a
bsolute phrases could be written with the word being in them, more formal Englis
h (and ETS!) tends not to use being when being is optional. If you ve studied GM
AT Sentence Correction for a while, then you know that the word being raises a b
ig red flag on the test! Here are some examples: • • The movie being over, we left t
he theater. This sentence could be rewritten like this: The movie over, we left
the theater. Similarly, having + past participle is often so semantically simila
r to the sentence without it that many sentences are written without having + pa
st participle.
An example would be very good here: • Having been chosen to head the committee, An
gus Ng thought about how he could help raise money for his chess club at Harvard
. This sentence could look like this: • Chosen to head the committee, Angus Ng tho
ught about how he could help raise money for his chess club at Harvard. This con
cept is important for the Sentence Correction section of the GMAT, so if you re
preparing for that test, pay attention to this!
3. noun + adjective Another pattern is to use an adjective after the noun it mod
ifies. Look at these examples: • • • Their meal still not ready after 45 minutes, the
hungry and angry customers left the restaurant. His hat in hand and pride in che
ck, Horace asked his former boss for his job back. The previews still showing, K
elly and Chris decided to leave the theater and enjoy the sunny day. 3) Parallel
Construction a. Is there a list? b. Is the sentence in two parts? -Both types m
ust have parallel types of verbiage. i.e. to ____, to ____ or ate _____, slept _
___, drank ____. Bad construction might look like to ____, _____ or ate _____, s
leep _____, drank ____. c. Comparisons must be logical • Find the two things being
compared and see if the sentence is structured in balance. Don’t be afraid to con
sider changing verbs or adjectives to get the balance. d. Parallelism is not jus
t about clauses, but verb usage Example: -ing and –ing, to…… to…… , either ….. or, neither
or.
e. The words "like," "unlike," "similar to," "as…so", “when” and "in contrast to"
are the most common indicators of comparisons. In comparisons, compatibility is
determined by subject matter. For example: As domesticated animals, indoor cats
typically lose their ability to hunt for their own food, so too do domesticated
dogs come to rely exclusively on their owners for sustenance.
Here, domesticated cats are compared to domesticated dogs, and the comparison wo
rks because they are both domesticated animals — they are like terms. Whenever you
see a comparison being set up in a sentence, check to see that the terms of the
comparison are compatible. 4) Verb Tense a. Is the whole sentence in the same t
ense b. Some major categories of tense: Don’t need to memorize types. Just be fami
liar Present tense Simple Past miles a day. Present Perfect Past Perfect Future
example: He has walked. example: He had walked. example: He will work. example:
He walks three miles a day. example: When he was younger, he walked three
Present Perfect – Describes action that began in the past but continues until the
present Key identifier – “has” “have” Sidenote: Sometimes used when deadline exists. Past
Perfect – Describes action that started and stopped in the past Key identifier – “had” P
resent Progressive – Used as emphasis by the speaker that the action is happening
this very minute Key identifier – verb ‘to be’ + a verb with an –ing ending. Perfect Pro
gressive – Occupies more than one moment in the past. In other words, ongoing for
a period of time. Key identifier – “had been” c. Two events that have taken place , ar
e taking place or will take place at the same time must have the same tense in t
he sentence. d. Passive verbs begin with the form of “to be” (Example: to be, were,
was) and end with a different verb in the past tense. 5) Subject-Verb agreement
errors. a. Singular + Plural agreement. Do the two agree in plurality? Can be ma
de very complex when prepositional phrases separate verb from noun/subject by 5
or more words. Easy to overlook cross referenced subject-verb relationship. b. T
o tell if a verb paradigm is plural or singular.
1) Mentally put “They” in front of the verb ß plural 2) Mentally put “He” in front of the
verb ß singular
c. Verb plurality: (This comes up A LOT on the GMAT) Adding an “s” to the end of an
adjective makes it singular. For example, dislike ß plural 6) Parallelism (Apples
+ Oranges) a. When the sentence compares two items. Ask yourself, can they be re
ally compared? b. When the sentence compares two actions as well. Usually, the p
roblem is with hidden comparison where two things or actions are compared, but a
nother two items or actions are intertwined and you lose the comparison relation
ship. Example: Synthetic oils burn less efficiently than natural oils. The sente
nce is wrong because we are trying to compare well each oil burns and not the oi
ls themselves. But do you see how the actual thing being compared is easily miss
ed? In a series of two or more elements, what you do on #2 determines what you d
o on 3+. In other words, everything after #2 must match #2: • • are okay. • • I like to
swim, run, and to dance. I like to swim, to run, and dance. are NOT okay. 7) Qua
ntity Words a. The words measuring quantity may be used incorrectly. For example
, when comparing two items, it would be inappropriate to use “among” to compare them
. Here’s a chart: 2 items Between More Better Less can’t say “fewer soup”. Here’s a chart:
Ok words for non-countable Less Amount, quantity Much Countable words fewer num
ber many if more than 2 among most best least I like to swim, to run, and to dan
ce. I like to swim, run, and dance. dislikes ß singular
b. Items that can’t be counted should not use quantity words. For example, you
c.
When two distinct words or phrases are joined by the correlatives either, or, ne
ither, nor, not only, but also, the number( singular or plural) of the word or p
hrase nearest to the verb determines the number of the verb. 1) Example: Either
his parents or he is bringing it (notice “is” is singular) This can be a confusing s
entence because parents is plural, but we pay attention to he which is the noun “h
e” tells us that we need to keep “is” singular. 2) Example: Either he or his parents a
re bringing it. Notice “ parents” is plural and is the closest to the verb so we use
“are” which is plural.
8) Idioms • • Definition: Incorrect usage of idiomatic expressions. There are no rul
es. Really need good English familiarity. Overall rule: If it’s not one of the pre
vious 7, then it’s very likely an idiom expression test question. Major Idioms you
should be pretty familiar with. a debate over a lot a responsibility to a resul
t of a sequence of agree with as an instance of as good as/or better than as gre
at as as good as...or better than as much as attend to (someone) attribute X to
Y/X is attributed to Y based on believe X to be Y both X and Y centers on concer
ned with conform to created with defined as depends on whether depicted as enabl
e to fascinated by forbid X to do Y identical with had better(do) in contrast to
independent from indifferent towards modeled after (no) more...than/(no) less..
.than more than ever must have (done) neither...nor not only...but also not so m
uch...as prohibits X from doing Y potential to range from X to Y reason….. that re
gard as regardless responsible for resulting in retroactive to incorrectly seen
as reason….. because
different from/differ from distinguishes between X and Y distinguish from doubt
that either...or
so X as to be Y so (adjective) that subscribe to such...as the same to X as to Y
to …….. used to (example to get used to or to become used to) to contrast X with Y
to mistake X for Y to result in to sacrifice X for Y used to (do)
1) estimated ...to be ...(SC-14,pg 699 of OG) 2) distinguishes between X and Y (
SC-15,pg 699 of OG) 3) one attributes X (an effect) to Y (a cause) (SC-21,pg 701
of OG) 4) X is attributed to Y (SC-21,pg 701 of OG) 5) not in a flash...but in
a ... (SC-22,pg 701 of OG) 6) affect to.. (SC-47,pg 707 of OG) 7) to orbit...(SC
-49,pg 708 of OG) 8) same to X as to Y (sc-54,pg 709 of OG) 9) extent to ...(SC-
57,pg 710 of OG) 10) range from X to Y ...(SC-58,pg 710 of OG) 11) range of ...(
SC-77,pg 714 of OG) 12) One X for every ZZ( some numeric number) Y s ...(SC-89,p
g 717 of OG) 13) seem to indicate ...(SC-95,pg 718 of OG) 14) more...than ever..
.(SC-98,pg 718 of OG) 15) X forbids Y to do Z ...(SC-100,pg 719 of OG) 16) X pro
hobits Y from doing Z ...(SC-100,pg 719 of OG) 17) as much as..(SC-105,pg 720 of
OG) 18) for jobs..(SC-109,pg 721 of OG) 19) credited with ..(SC-111,pg 721 of O
G) 20) research to (SC-112, pg 721 of OG) 21) between X and Y (Sc-115, pg 722 of
OG) 22) better served by X than Y ..(SC-116,pg 722 of OG) 23) X ordered Y to do
Z (sc-121,pg 724 of OG) 24) restitution...for ...(SC-130,pg 726 of OG) 25) X [i
s] expected to Y (SC-131, pg 726 of OG) 26) viewed marriage as ( sc-147,pg 730 o
f OG) 27) to mistake X for Y (sc-150,pg 730 of OG) 28) not X ...but rather Y ..(
SC-172,pg 736 of OG)
29) persuaded X to do Y (sc-173,pg 736 of OG) 30) more X than Y ...(SC-175,pg 73
7 of OG) 31) so X that Y ...(SC-177,pg 737 of OG) 32) to survive (SC-181,pg 738
of OG) 33) X as Y ..(SC-182,pg 738 of OG) 34) targeted at ( SC-195,pg 742 of OG)
35) X ordered Y to be Z ed..(SC-218,pg 748 of OG) 36) X ordered that Y be Z ed.
..(SC-218,pg 748) 37) interaction of ...(SC-237,pg 753 of OG) 38) to monitor ...
(SC-239,pg 754 of OG) 39) that X ...that Y ...(SC-250,pg 757 of OG) 40) noted th
at ..(Sc-258,pg 759 of OG) 41) as a result of...(SC-263,pg 761 of OG) 42) call..
.to consider... 43) no less....than 44) believe X to be Y .... 45) from X to Y .
.. 46) entrusted with... 47) accompanied by.... 48) decline in.... 49) benefit f
rom... 50) alternative to.... 51) regard...as... 52) expected X to be Y ... 53)
expected that X would be Y ... 54) contrary to... 55) widely anticipated that...
. 56) modeled after.... 57) for over...XXX years... 58) assume ...to be of... 59
) seem...to...(seem is plural) 60) used in the construction... 61) used to const
ruct... 62) replacing with... 63) regards X as Y ... 64) forcing ...to... There
are three types of idioms that you ll see on the GMAT: 1. 2. 3. word pairs that
go together prepositions and the verbs that use them standard expressions.
Word Pairs as…as more…than less…than greater….than not only…but (also) so…that She was not
nly exhausted but (also) famished as well. The apartment was so expensive that n
o self-supporting student could afford it. (Just) as it is the duty of employees
to contribute to the well(just) as…so neither…nor either…or being of the company, so
it is the duty of the company to contribute to the well-being of its employees.
Neither a borrower nor a lender be. The workshop was more thrilling than anythin
g I d ever done. The movie was as long as it was boring.
Idiomatic preposition usage Many idiomatic expressions tested on the exam involv
e prepositions. There s no overarching grammatical rule that tells you which pre
positions go with which verbs. Again, the rules are determined by usage, so you
ll have to "listen" to the expression and determine if the verb is followed by t
he correct preposition. Be on the lookout for commonly tested prepositions like
"of," "at," "by," "in," "from," "to," and "for". If you have difficulty determin
ing whether a usage is correct, try testing out the idiom in a simpler version o
f the sentence. Example: Although he was considered as a leading proponent for t
he controversial new initiative, the professor nevertheless sought refuge from t
he media uproar. The sentence above becomes: The professor was considered as a p
roponent. Does anything sound unusual? Could this sentence be worded differently
? In fact, to be, not as, is the correct idiom: The professor was considered to
be a proponent. Would rather ß present tense if referring to myself and past tense
if someone other than the subject is doing the action Example: Would rather I s
peak ß present tense Example: Would rather you spoke ß past tense General Grammar de
finitions So= therefore So that=in order to/in order that That=the fact that
Conjunctions that can join two independent clauses are: and, but, yet, for, or,
nor Don’t start sentences with “Because” Words requiring “how” (example: know how) Know Te
ach Learn Show Always choose active over passive voice Active example: Elaine pu
rchased new software for the company Passive example: New software was purchased
for the company by Elaine Another frequently tested grammatical error is unnece
ssary use of the passive voice. It s a good idea to become familiar with this ty
pe of error; it appears quite often in the answer choices of sentence correction
questions. As a reminder, the passive voice is in use when the action of the se
ntence is performed on the subject. The active voice is in use when the subject
itself performs the action. Example: Active: Allison (subject) went (action) to
the store to buy a cake (object). Passive: The cake (subject) was bought (action
) by Allison (object). Preposition use “To” or “Of” needs subject + verb NOT object + ve
rb Examples of subject: she, he, whoever, who, I Examples of object: her, him, w
homever, its, it Either side of a form “to be” (were, was) must have subjects not ob
jects on either side to agree. A verb that ends in –ing is a gerund which is a ver
b acting like a noun. Try not to use gerunds if you can help it. Adverbs, not ad
jectives modify verbs Key identifier: Adverbs end in –ly Correct example: I sure w
ish I were rich Incorrect example: I surely wish I were rich Word Usage
Some pairs of words, like fewer and less, are often used incorrectly because the
y re treated as synonyms. In fact, there is a solid rule that determines which o
ne you should use, and the exam will test your ability to decide which is the co
rrect option. The exam tests four such word pairs with particular frequency, so
memorize the rules pertaining to them if you don t know them already. 1. Fewer/L
ess Which of the two following sentences is correct? 1. 2. The company fired no
less than fifty employees. The company fired no fewer than fifty employees.
The second sentence is correct. Why? Because you use less when you re talking ab
out things you can t count (less pollution, less violence) but fewer when you re
talking about things you can count (fewer pollutants, fewer violent acts). 2. N
umber/Amount These words follow the same rule as less and fewer. Number is corre
ct when you can count the thing being described (a number of cars, a number of p
eople) and amount is correct when you cannot (amount of love, amount of pain). 3
. Among/Between Use between when only two options are available (between the red
car and the blue car) and among when more than two options are available (among
the five answer choices, among the many books). 4. If/Whether Whether is correc
t when you re discussing two options (whether to get chocolate or strawberry ice
cream) and if is correct for more than two options (if she should get ice cream
, frozen yogurt, or a cookie). The use of being People who study for GMAT for a
while quickly learn that being is usually wrong. So I m guessing you already kno
w that being in an answer choice is wrong more often than it is right. This is a
good strategy to get you started, but to get over 700 on the GMAT, you really n
eed to know some of the finer points of GMAT Sentence Correction that relate to
the use of the word being.
There are at least two different situations in which being is often the right an
swer. Here is the first example of when being is correct: When the grammar requi
res it. Yes, I m trying to simplify things here, but the idea is this--many idea
s can be expressed in more than one way. For example, I can say: I m afraid of b
eing late. I m afraid that I ll be late. Each has its own emphasis, but the poin
t is that these two structures exist. (If I know my members here, I know that th
ey will have questions about the difference, but please, let s start a separate
thread for this.) Whether we can express ideas in one or more structures is real
ly related to the word used; in other words, it is idiomatic. But some idioms al
low only one structure. For example: In addition to being one of the first resta
urants to combine Mediterranean and American tastes, Chez Panisse in Berkeley is
also one of the Bay Area s most established restaurants. The idiomatic structur
e in addition to does not have a counterpart that uses a subject and a verb, so
our only option here is to use being, which is grammatically a noun, but is deri
ved from a verb. The second example of when being is correct is shown in this ex
ample: There are many reasons to get an MBA, with increased career prospects bei
ng the most important for many MBA applicants. Technically this part here: with
increased career prospects being the most important for many MBA applicants
is an absolute phrase, but I think it s also helpful just to memorize the patter
n: with + NOUN + being + NOUN COMPLEMENT
Like vs. Such As Question: What s the difference between like and such as? Examp
le of the "mistake" that we make in everyday speech: Can you buy me some fruit l
ike oranges or grapefruit? How the GMAT Official Guide would explain this mistak
e: Using like in this answer choice mistakenly suggests that the utterer of the
request does in fact not want oranges or grapefruit, but rather some other kind
of fruit that is similar to oranges or grapefruit. In normal English: In GMATLan
d, like means similar to, and such as means for example. Take a look at these ex
amples: • Can you buy me some fruit like oranges or grapefruit? In GMATLand, this
sentence would mean that you do NOT want oranges or grapefruit; instead, you d p
refer some fruit similar to oranges and grapefruit. For example, you may want po
melo, lemons, or limes. Yes, I know this sounds a little crazy, but our goal is
to understand what GMAT is looking for, not what is "correct" English. • Can you b
uy me some fruit such as oranges or grapefruit? Yes, this is what we re supposed
to say in GMATLand -- oranges and grapefruit are examples of the type of fruit
we want. • I would like you to buy such fruit as oranges and grapefruit for me, if
you don t mind. This is simply a variation -- notice how such and as are separa
ted. Separating the two elements tends to make this pattern a bit harder to see.
Subject/Verb Inversion So you already know that the GMAT test is an adaptive te
st, meaning that your score goes up or down depending on the difficulty of the q
uestions that you answer correctly or incorrectly. On the sentence correction se
ction of the GMAT, the questions that test you on subject/verb inversion tend to
be the harder questions, and are therefore worth more points. So, to raise your
GMAT score, you should be very familiar with most or all of the items on this l
ist! I ve spent a few years developing this list, and feel confident that it rep
resents most or all of the inversion points that you ll see on the GMAT, TOEFL,
or SAT II: Writing tests. There are at least eighteen types of inversion: 1. neg
intro 2. intro adverbial (in, down, prepositional phrase) 3. intro -ed 4. compa
rative 5. intro comparative
6. as 7. so... that... 8. had, should, were 9. there is 10. here is 11. intro -i
ng 12. emphasis 13. the bigger, the better 14. questions 15. "story speech" 16.
nor 17. so do I/neither do I 18. intro adjective
Type 1. neg intro
Examples Never do I sleep. Only at night can I study. In no way could I help you
with your Japanese grammar question. I believe that only rarely will I need you
r help. Not until I got home did I realize that my shoes were untied.
Notes Question form is obligatory. Used with all verbs. This one is very common
on the TOEFL and somewhat common on the GMAT and GRE. We need to learn the vario
us types of words and phrases that require this type of inversion. Notice that s
ometimes the inversion occurs right after the neg intro form and sometimes it oc
curs in the next subject and verb. See Neg Intro for more info.
2. intro adverbial
Into the room ran the lady. First comes love, then comes marriage. After A comes
B, then comes C, next comes D. Down came the rain and washed the spider out.
Inversion is optional. Used with be-verbs, linking verbs, and verbs of direction
. This one is less common on the TOEFL, but more common on the GMAT and GRE. Not
ice that sometimes we have an adverb, like first and down and sometimes we have
an adverb phrase like into the room or after A.
Type
Examples
Notes These adverbs and adverb phrases usually show location or direction. This
type of inversion usually only occurs with be-verbs, linking verbs and verbs tha
t show direction or movement, like come, go, run, etc.
3. intro –ed
Found in San Francisco is Lombard Street, the so-called crookedest street in the
world. Lost among the old tables and chairs was the priceless Victorian desk. L
ocated between San Francisco and Marin County is the Golden Gate Bridge.
Inversion is obligatory. Used with be-verbs. This one is very common on the TOEF
L, GMAT, and GRE. This type of inversion usually occurs with be-verbs, but somet
imes with linking verbs. Notice that the phrase is the complement of the be-verb
. Inversion is optional. Used with all verbs. This form of inversion is common o
n the TOEFL, GMAT, and GRE. We normally only have inversion here if we are compa
ring subjects of the verb, not objects. For example, in the following two senten
ces, we are comparing objects, carrots and potatoes, not the subject I.: ϑ ϑ I like
carrots more than I do potatoes. Λ Λ I like carrots more than do I like potatoes. No
w, in this sentence, we are comparing subjects, I and my friend Carl: ϑ ϑ I like car
rots more than does my friend Carl.
4. comparatives

Cheetahs run faster than do antelopes. You speak Chinese better than do I. essi
ca is more interested in Computer Science than is Benjamin.
5. intro
Bigger than an apatosaur is
Inversion is obligatory.
Type comparative
Examples the blue whale. More important than your personal statement is your GPA
. No less impressive than the invention of the laser was the development of the
wheel.
Notes Used with
 be-verbs. This form is more common on the GMAT and GRE than it i
s on the TOEF . Notice that we can only use this form of inversion when the verb
is a be-verb since in every case, the comparative is the complement of the be-v
erb. Remember that less than is also a comparative.
6. as

Megumi is from apan, as is Sato. So-eun wants to leave early today, as does Oi.
If thrown into the water, camels can swim, as can cats.
Inversion is obligatory. Used with all verbs. We can only use inversion if we ar
e using as for comparisons. as is one of the trickiest words in English; it can
have many different meanings.
7. so… that…
So happy was I that I bought flowers for everybody in class. So quickly did she
leave that we did not even realize was gone. So rarely does a comet appear visib
le to the naked eye that when one does, it is considered a major event.
Question
 form is obligatory. Used with all verbs. This is not so common on the T
OEF , but is fairly common on the GMAT and GRE. The so… that… clause must before the
verb in for this type of inversion.
8. had, should, were for ifclauses
Had I remembered Tomomi’s birthday, she wouldn’t be mad at me now. Should you need a
hand, I will be more than happy to help you. Were I you, I think I would study
more for your exam tomorrow.
Inversion
 is obligatory. Used with all verbs. This is somewhat common on the TOE
F and more common on the GMAT and GRE. This type of inversion is kind of specia
l. Notice that we can only use this type of inversion when we are using an if-cl
ause. In other
Type
Examples
Notes words, if is omitted: even though the word if does not appear in the claus
e, we still have the meaning of an if-clause. For more information, see had, sho
uld, were.
9. there is, there are, there exists, there comes, etc.
There is a good restaurant nearby. There comes a time in every person’s life when
she realizes that she is responsible for her own happiness, not other people. Sc
ientists hypothesize that there exists a certain type of particle that can trave
l faster than the speed of light.
Inversion is obligatory. Usually used only with these verbs. This form of invers
ion is common on the TOEF , GMAT, and GRE, as well as in spoken and written Engl
ish. Most people remember there is and there are. BUT we must also remember that
there are other verbs that we can use instead of is and are. The most common on
es are exist, come, and go.
10. here is, here are, here comes, here come
Here is some good food for you to try. Here are the books that I don’t need anymor
e. Here comes the bus!
Inversion is obligatory. Usually used only with
 these verbs. You will probably n
ot see this on the grammar
 section of the TOEF or on the GMAT or GRE.
 It could,
however, appear on the istening Comprehension Section of the TOEF . We use thi
s form mostly in spoken English.
11. intro -ing
Burning out of control was the forest located in
 the foothills
 of the Sierra Nev
ada mountains. Coming in last in the race was oe “Elephant egs” Blow. Not helping
the situation was little Susie, who was throwing newspaper on the spreading fire
.
Inversion
 is obligatory. Used only with be-verbs. This form is not common on the
TOEF , but might show up on the GMAT or GRE. Notice the intro –ing phrase is the
complement of the be-verb.
Type 12. emphasis
Examples Boy am I hungry. Is it ever hot in here! Do you know how to cook!
Notes Inversion is optional. Used with
 all verbs. You will probably not see this
on the grammar
 section of the TOEF or on the GMAT or GRE.
 It could, however, a
ppear on the istening Comprehension Section of the TOEF . We use this form most
ly in spoken English.
13. the bigger, the better
The closer an object is to another object, the greater is the gravity between th
e two objects.
Question form is optional. Used with all verbs.
14. questions
Is this the last example? Do you enjoy reading these lists? Are we finished yet?
Inversion is obligatory. Used with
 allverbs. You will probably not see this on
the grammar section of the TOEF (TOEF doesn’t  test questions anymore) or on the
GMAT or GRE.
 It would, however, appear on the istening Comprehension Section of
the TOEF .
15. "story speech"
“I think it’s time to go,” said Susan. “It’s time for you, but not for me,” replied Gary. “
be we should collect our thoughts for a moment,” commented any.
Inversion is optional. Used with verbs that report speech. You will probably not
see this on the grammar section of the TOEF or on the GMAT or GRE.
16. nor
No one has volunteered for the job, nor do we expect
 anyone to volunteer in the
future. Hok-ming cannot speak Portuguese, nor can osé speak Cantonese.
Inversion
 is obligatory. Used with all verbs. You might see this on the adaptive
TOEF if you are scoring high and it could appear on the GMAT or GRE.
Type
Examples The zoo regulations will not permit you to touch the animals, nor would
most people advise you to do so.
Notes Remember that nor is considered a conjunction, but we use it between two s
entences (not between any two elements like the other conjunctions).
17. "so do I"/ "neither do I."
“So do I.” “So can Terry.” “Neither do most people I know.”
Inversion is obligatory. Used with
 all verbs. You will probably not see this on
the grammar section of the TOEF or on the GMAT or GRE.
18. intro adjective
Beautiful beyond belief was
 my baby daughter. Happy about their acceptance into
their dream schools were any and Tomo. Quick and painless will be your medical
procedure.
Inversion is obligatory in most cases. Used with be-verbs. This one is fairly ra
re and probably would not appear on the TOEF , but you might see it on the GMAT
or GRE. Inversion is sometimes not used in poetic language.
What does Question form is obligatory mean? This simply means that you MUST inve
rt the subject and the verb in this construction. In other constructions, invers
ion is optional, but in these constructions, it is required. For example, you ma
y say:
• She runs faster than do most of her classmates.
(verb comes before the subject) or
• She runs faster than most of her classmates do.
(subject comes before the verb) BUT
• Never have I heard such a thing!
CANNOT become

• XX Never I have heard such a thing! XX


ike vs. Such As Question: What s the difference between like and such as?
Example of the "mistake" that we make in everyday speech: Can you buy me some fr
uit like oranges or grapefruit? How the GMAT Official Guide would explain this m
istake: Using like in this answer choice mistakenly suggests that the utterer of
the request does in fact not want oranges or grapefruit, but rather some other
kind
 of fruit that is similar to oranges or grapefruit. In normal English: In GM
AT and, like means similar to, and such as means for example. Take a look atthe
se examples: • Can you buy me some fruit like oranges or grapefruit? In GMAT and,
this sentence would mean that you do NOT want oranges or grapefruit; instead, yo
u d prefer some fruit similar to oranges and grapefruit. For example, you may wa
nt pomelo, lemons, or limes. Yes, I know this sounds a little crazy, but our goa
l is to understand what GMAT is looking for, not what is "correct" English. • Can
you buy me some fruit
 such as oranges or grapefruit? Yes, this is what we re sup
posed to say in GMAT and -- oranges and grapefruit are examples of the type of f
ruit we want. • I would like you to buy such fruit as oranges and grapefruit for m
e, if you don t mind. This is simply a variation -- notice how such and as are s
eparated. Separating the two elements tends to make this pattern a bit harder to
see. What s the Difference Between that and which? After people study GMAT sent
ence correction for a while, they may ask about the difference between that and
which. I have to say for the record, though, that it is not terribly important t
o know the difference between the two to get a high score on the test, but I kno
w that some people are naturally curious about the difference (people here tend
to be overachievers!). A Tip But first, a testmagic tip: GMAT almost always (I s
ay almost always because I ve seen two questions that did not follow this rule,
but the rule was violated in all five answer choices) wants you to put a comma b
efore which. In other words, if you see which without a comma before it, it s pr
obably wrong. After this explanation, you should understand why, but for those o
f you who want only the most important information, this is what you need to kno
w. An Example Both of these sentences are correct in GMAT land: • • Please go into t
he room and get me the big book, which is mine. Please go into the room and get
me the big book that is mine.
Yes, in GMATland, these two sentences have two different meanings. Both of these
sentences would be incorrect in GMAT land: • • X Please go into the room and get me
the big book which is mine. X X Please go into the room and get me the big book
, that is mine. X
Notice the commas--that s what makes all the difference. The Explanation
Okay, we have in English this weird idea that we need to use different grammar i
n an adjective clause (a.k.a. relative clause) depending on whether the informat
ion in the adjective clause is necessary to specifically identify which noun we
are referring to. For example, imagine you have one sister, and you are telling
a friend that your sister is coming to visit you. Since this person is your frie
nd, we can presume that he knows that you have only one sister. You utter a sent
ence like this to your friend in GMATland: • "My sister, who just graduated from c
ollege, is coming to see me." In GMATland, since your friend (we presume) knows
you well and knows that you have only one sister, this extra bit of information
is considered unnecessary to identify which sister it is you are talking about.
It is a sort of "by the way" information--"My sister is coming to see me, and oh
, by the way, she just graduated from college." Now imagine you have two, three,
or even more sisters. et s imagine that one is a college professor, another is
a webmaster, and this one who is coming to visit you just graduated from colleg
e. If you re talking to your friend, and you say only "my sister," and you do no
t mention her name, your friend might not know which sister you are talking abou
t. So you add that extra bit of information--my sister who just graduated from c
ollege--to identify which sister it is you are referring to. In this situation,
we have just correctly employed a very important grammar rule. Read on. So, if t
he person you re talking to, or the person who s reading what you ve written, ne
eds that extra bit of information to know which noun you re referring to, we say
that that extra information is non-restrictive. This word doesn t really descri
be the function clearly, so many teachers say that this information is "extra."
On the other hand, if you need that information to know which noun you are talki
ng about, we say that the information is restrictive. Again, this word is not re
ally a good choice for clarity, and many teachers use the term "necessary inform
ation" instead. Finally, just to make English a bit more difficult, we have a ru
le that says we should use a comma before or after "extra information clauses an
d phrases," but not with "necessary information clauses or phrases." The idea he
re is that the comma represents the slight pause in speech or change in intonati
on that a native speaker might use when making such an utterance. I should point
out that both that and which are relative pronouns, i.e., they are grammaticall
y the same, but their meanings are slightly different. Now, let s return to our
original example sentences: • Please go into the room and get me the big book, whi
ch is mine. In this sentence, the clause which is mine is "extra" because the in
formation "the big book" is enough to identify which book it is that you want. W
e can assume that there is only one big book in the room. • Please go into the roo
m and get me the big book that is mine.
In this sentence, the clause that is mine is "necessary" because the information
"the big book" is NOT enough to identify which book it is that you want--it is
probably the case that there are several big books in the room, so I need to add
the information "that is mine" to identify which book it is that I want. More E
xamples A few more examples may help: • I met with Bill Clinton, who is a lawyer.
The name Bill Clinton is enough to identify which person I m talking about--who
is a lawyer is therefore extra information. • I met with the man who is a lawyer.
In this case, "the man" is not enough information to identify which person I m t
alking about-who is a lawyer is therefore necessary information. • The Sun, which
is the only star in our solar system, is the source of heat for Earth. Again, th
e name "the Sun" already clearly identifies the noun; therefore, the information
in the adjective clause "which is the only star in our solar system" is extra. •
The star that is at the center of our solar system is called what? In this case,
since we don t have a name here, we don t know which star it is that we are ref
erring to. Therefore, the information in the adjective clause "that is at the ce
nter our solar system" is necessary. A good rule to learn is that which requires
a comma before it. If you re not sure about why the comma is necessary, please
see this post. However, GMAT is very tricky at times, and can of course create a
question in which this rule doesNOT apply. In fact, I ve seen a question that
used which without a comma, but A the answer choices violated the rule. I.e.,
none of the answer choices used
 that; they all used which without a comma. Here
s an example of what I mean: ucise in San Francisco is a place which anybody ca
n visit. (A) which anybody can visit (B) which any person could visit (C) which
no person could not visit (D) which, if they wanted to, any person could visit (
E) which any person could visit if they so desired I know some of you will think
that this sentence is terrible, but the whole point of GMAT sentence correction
is that we must choose the best answer, NOT the perfect answer. Critical Reason
ing
On verbal part of the GMAT, you will encounter about 14 Critical Reasoning quest
ions of various lengths (sometimes you will even need to scroll to read all the
answer choices). In 99 per cent of cases, you will have a short passage with one
question right under it. The argument you meet can be anything from a classical
argument to an advertisement or a dialog. The questions will ask you to manipul
ate the argument to weaken/strengthen it, find the conclusion, assumption, expla
nation, do an inference, supplement a statement, or even tell how its parts are
related to each other. On average, you will have 1:50 for each question, but it
is recommended that you try to stay within 1:30 on CR (Critical Reasoning) quest
ions since you will need to save some time for Reading Comprehension. It is reco
mmended that you read through Kaplan s Verbal workbook or the Section of CR in t
he Kaplan GMAT book with CD, 5th edition. Both of the books are good for buildin
g a solid background; in our sessions, we will develop further the techniques de
scribed in these books, yet will not rely on them for exercises or anything else
. First of all, Critical Reasoning is ability to reason clearly to evaluate and
judge arguments. You are using this skill a lot during you everyday life while r
eading newspapers or watching movies. When you think that the movie is pushing t
he limit of the Reasonable or the news sounds less reasonable than the movie tha
t was pushing the limit, you are using your CR skills to produce these conclusio
ns. Besides the Verbal part on the GMAT, you will also need good argumentative s
kills to beat the essays since one of them is to construct an argument, and the
other is to evaluate one. As a rule, GMAT CR questions will ask you to manipulat
e the argument to weaken/strengthen it, find the conclusion, assumption, explana
tion, do an inference or supplement a statement, etc. Whatever it is that you ha
ve to do, you will need 2 things to succeed: know the basic structure of argumen
ts and clearly understand the argument. In general, about 80% of GMAT arguments
consist of evidence, usually 2 pieces, a conclusion - the main point of an argum
ent, and an assumption - the bridge between the evidence and conclusion. The maj
ority of the arguments you encounter on the test will be 3 step arguments: Evide
nce1 + Evidence2 = Conclusion.

Example 1. ast week Mike was detained for shoplifting at a groceries store near
his house, but he has been a Christian for 10 years, therefore, the police must
have been wrong accusing him in stealing. We have here two pieces of evidence:
Mike was accused of stealing and that his is a Christian. The conclusion is that
the police are wrong. Therefore, our huge assumption here is that a Christian c
ould not have stolen anything. Example 2. There are a lot of mosquitoes outside
today, please do not turn on the light in the room because a lot of them will fl
y in. There is no set scheme for structure in GMAT CR, but since the majority of
the arguments are only a few sentences long, the conclusion usually comes in th
e first or the last sentence. However, some of the arguments you
 encounter will
not have a conclusion at all or will have just an implied one. et s go through
the strategy to approach CR questions; we will cover it today in general and the
n will practice some of the crucial steps one by one and then in the third sessi
on will consolidate the approach. Critical Reasoning Strategy [We want to warn y
ou immediately that this strategy is not the easiest way to do CR (the easiest w
ould be read-and-answer), but it lets you get the most questions right spending
less time per correct answer. The bottom line is that it won t be easy to follow
this strategy but if you do, it will reward you] 1. Read the question (this nee
ded so that you would know what to look for and what to do: find an assumption,
weaken, infer something or else; do not worry about the details in the question,
read for keywords, such as strengthen, deny, or explain.
Sometimes at the end of the Verbal section on the test, your brain won t even ho
ld a keyword, so you may want to write it down symbolically; e.g. + for strength
en or - for weaken. 2. Read the passage (Read it very attentively because in con
trast to Reading Comprehension, there is very little text here and mostly everyt
hing is important; try to read only once. Reread only hard texts). 3. As you rea
d, look for the problem in the passage (evaluate how convincing it is) 4. Paraph
rase the passage (this a very important step because when you do a paraphrase, y
ou check whether you understood the passage and at the same time you extract the
skeleton of the argument, making it easier to identify the conclusion and the a
ssumption. Very often, the paraphrase of the passage will be pretty close to the
conclusion. It is not surprising, since the conclusion is the main point and ev
idence just supports it.) Your paraphrase should be as close to the text and as
simple as possible so that you would understand it easily and at the same time c
ould fully trust it. Do not make it too general nor too detail oriented. When yo
u do a paraphrase, do it in three steps: Evidence1, Evidence2, and Conclusion; p
ut "therefore" word before you start your conclusion, this will help you to set
it off. 5. Read the question again (now with more understanding of what is being
asked; reading the question 2 times, will also help you to make sure you answer
exactly what is stated and that you understand the question.) 6. Answer before
reading the answer choices (Why do this? Two reasons: one, if you can think of t
he correct answer or at least the general direction that the answer choice needs
to be, you will identify it among the wrong choices much faster, thus spend les
s time reading the answers, which usually take 30 seconds to cover. The second r
eason is that often test takers are seduced by the author s wording. One reads a
few words that were used in the passage and the brain identifies this choice wi
th the passage, thus making it seem more right that it needs to be. The more pro
blems you practice with, the more chance is you will guess the right answer even
before reading it. And there is nothing more pleasant than seeing YOUR answer c
hoices among the listed. ) 7. Go through the answers, first time scan them for Y
OUR answer choice (usually you will guess correctly in 60-70% of cases), if you
did not find it, reread them more attentively. 8. Draw a grid to eliminate the w
rong answers easier. Use "+" for a sure answer, "-" for a definitely wrong answe
r choice, and "~" or "?" for an answer that may be right or questionable. This w
ill help to concentrate only on a few answer choices and will prevent you from r
eading same answers several times if you get confused or keep having troubles lo
cating the right answer. A B C ? + -

D E + 9. Always remember to think how the answer choice relates EXACT Y to this
situation; it may be out of scope by being too general. E.g. Advertisement: For
sinus pain, three out of four hospitals give their patients Novex. So when you w
ant the most effective painkiller for sinus pain, Novex is the one to choose. Wh
ich of the following, if true, most seriously undermines the advertisement s arg
ument? (A) -(B) -(C) Many drug manufacturers increase sales of their products to
hospitals by selling these products to the hospitals at the lowest price the ma
nufacturers can afford. (D) Unlike some competing brands of painkillers, Novex i
s available from pharmacies without a doctor s prescription. (E) -As about this
session we will be mostly concerned with one of the most important parts of a gm
at argument, Assumption. Many CR questions directly ask for an assumption or are
based on them, such as weaken and strengthen questions. Also, assumption of an
argument is one of the only parts that we can influence to destroy or solidify a
n argument; thus if we can disprove an assumption, we can negate the whole argum
ent because the conclusion will not make sense. On the other hand, if we can str
engthen our assumption, thus patch a possible hole, we will create a stronger ar
gument and our conclusion will be more credible. (Sometimes some of the evidence
is doubted or is amended to add new meaning and change the argument, but genera
lly, it is the assumption that is attacked to destroy an argument. In a case whe
n evidence is completed with more info, we can still say that it is the assumpti
on that the evidence we had was valid is being destroyed). Thus, to succeed in C
R, it is crucial that you are able to extract an assumption fast; it will save y
ou time on many questions. Practicing with assumptions does not take too long; 2
-4 hours and your skills will be on top. Also, you will be a better speaker and
a more critical writer if you are able to see assumptions of the writer who argu
es against you because as we have said, if you can kill an assumption, you will
damage the argument. For example, if we play with Example 1 and add a piece of e
vidence that says that Christians sometimes steal, we will not be able to say wi
th confidence that the police are wrong about Mike being a thief, since evidence
proves that Christians do steal. We will also cover one of the most important s
teps of the CR approach, paraphrase. Paraphrase allows you to check how well you
understood the passage as well as to see the
structure stripped. Paraphrase should not take you more than 5-10 seconds after
you have read the passage. Sometimes it is useful to write down some complicated
relationship or something that will help you to understand the passage better,
such as a diagram, for example. Other than that, don t write down anything else,
the passage that you read will be so short that you should be able to remember
all the details without writing down anything. Today we were supposed to learn t
hat the assumption of an argument is • • • • • • • • • • a bridge (link) between the eviden
e conclusion of an argument never stated in the text is the most vulnerable elem
ent of an argument is the only element in the argument that can be influenced sh
ould be easy for you to find by now Should be brief and take 5-10 seconds Will s
tate the main idea of the passage and be close to the conclusion Will help you t
o understand the passage better Will reveal the conclusion, evidence, and eventu
ally, the assumption Start your conclusion with Therefore
Paraphrase
Homework Work on the section 10 and 11 of the GMAT+ materials available for down
load at www.angelfire.com/scifi2/gmatplus/gmat.html, paraphrasing all passages a
nd writing down the assumptions. Also, please, underline the conclusions; some o
f the passages won t have a conclusion/assumption; in this case, leave out the c
onclusion and assumption, but still do a paraphrase. To check your answers, go t
o the appendix of the guide: 1) Section 10 can be found in Appendix B 2) Section
11 can be found in Appendix C Appendix A Free Scaled down version of informatio
n you’ll find in Deltacourse. Deltacourse is 10 times more thorough and covers mor
e subcategories of Quant question types not covered well in Princeton or Kaplan.
Arranging Objects The number of ways of arranging n unlike objects in a line is
n! (pronounced ‘n factorial’). n! = n × (n – 1) × (n – 2) ×…× 3 × 2 × 1 Example How many d
ys can the letters P, Q, R, S be arranged? The answer is 4! = 24.
This is because there are four spaces to be filled: _, _, _, _ The first space c
an be filled by any one of the four letters. The second space can be filled by a
ny of the remaining 3 letters. The third space can be filled by any of the 2 rem
aining letters and the final space must be filled by the one remaining letter. T
he total number of possible arrangements is therefore 4 × 3 × 2 × 1 = 4! The number of
ways of arranging n objects, of which p of one type are alike, q of a second ty
pe are alike, r of a third type are alike, etc is: n!/(p! q! r!) … Example In how
many ways can the letters in the word: STATISTICS be arranged? There are 3 S’s, 2
I’s and 3 T’s in this word, therefore, the number of ways of arranging the letters a
re: 10! = 50 400 3! 2! 3!
Rings and Roundabouts The number of ways of arranging n unlike objects in a ring
when clockwise and anticlockwise arrangements are different is (n – 1)! When cloc
kwise and anti-clockwise arrangements are the same, the number of ways is ½ (n – 1)!
Example Ten people go to a party. How many different ways can they be seated? A
nti-clockwise and clockwise arrangements are the same. Therefore, the total numb
er of ways is ½ (10-1)! = 181 440 Combinations The number of ways of selecting r o
bjects from n unlike objects is: nCr = n!/r! (n – r)! Example There are 10 balls i
n a bag numbered from 1 to 10. Three balls are selected at random. How many diff
erent ways are there of selecting the three balls? 10C3 = 10! = 10 × 9 × 8 = 120
3! (10 – 3)! 3 × 2 × 1
Permutations A permutation is an ordered arrangement. The number of ordered arra
ngements of r objects taken from n unlike objects is: nPr = n!/(n – r)! Example In
the Match of the Day’s goal of the month competition, you had to pick the top 3 g
oals out of 10. Since the order is important, it is the permutation formula whic
h we use. 10P3 = 10!/7! = 720 There are therefore 720 different ways of picking
the top three goals.
Probability The above facts
 can be used to help solve problems in probability. E
xample In the National ottery, 6 numbers are chosen from 49. You win if the 6 b
alls you pick match the sixballs selected by the machine. What is the probabili
ty of winning the National ottery? The number of ways of choosing 6 numbers fro
m 49 is 49C6 = 13 983 816 . Therefore the probability of winning the lottery is
1/13983816 = 0.000 000 071 5 (3sf), which is about a 1 in 14 million chance The
probability of an event occurring is the chance or likelihood of it occurring. T
he probability of an event A, written P(A), can be between zero and one, with P(
A) = 1 indicating that the event will certainly happen and with P(A) = 0 indicat
ing that event A will certainly not happen. Probability = the number of successf
ul outcomes of an experiment the number of possible outcomes So, for example, if
a coin were tossed, the probability of obtaining a head = ½, since there are 2 po
ssible outcomes (heads or tails) and 1 of these is the ‘successful’ outcome.
Using Set Notation Probability can be studied in conjunction with set theory, wi
th Venn Diagrams being particularly useful in analysis. The probability of a cer
tain event occurring, for example, can be represented by P(A). The probability o
f a different event occurring can be written P(B). Clearly, therefore, for two e
vents A and B, P(A) + P(B) - P(AnB) = P(AuB) P(AnB) represents the probability o
f A AND B occurring. P(AuB) represents the probability of A OR B occurring.
Mutual Exclusive Events Events A and B are mutually exclusive if they have no ev
ents in common. In other words, if A occurs B cannot occur and vice-versa. On a
Venn Diagram, this would mean that the circles representing events A and B would
not overlap. If, for example, we are asked to pick a card from a pack of 52, th
e probability
 that the card is red is ½ . The probability that the card is a club
is . However, if the card is red it can t be a club. These events are therefore m
utually exclusive. If two events are mutually exclusive, P(AnB) = 0, so P(A) + P
(B) = P(AuB)
Independent Events Two events are independent if the first one does not influenc
e the second. For example, if a bag contains 2 blue balls and 2 red balls and tw
o balls are selected randomly, the events are: a) independent if the first ball
is replaced after being selected b) not independent if the first ball is removed
without being replaced. In this instance, there are only three balls remaining
in the bag so the probabilities of selecting the various colours have changed. T
wo events are independent if (and only if): P(AnB) = P(A)P(B) This is known as t
he multiplication law.
Conditional Probability
Conditional probability is the probability of an event occurring,
 given that ano
ther event has occurred. For example, the probability of ohn doing mathematics
at A- evel, given that he is doing physics may be quite high. P(A|B) means the p
robability of A occurring, given that B has occurred. For two events A and B, P(
AnB) = P(A|B)P(B) and similarly P(AnB) = P(B|A)P(A). If two events are mutually
exclusive, then P(A|B) = 0 . Example A six-sided die is thrown. What is the prob
ability that the number thrown is prime, given that it is odd. The probability o
f obtaining an odd number is 3/6 = ½. Of these odd numbers, 2 of them are prime (3
and 5). P(prime | odd) = P(prime and odd) = 2/6 = 2/3 P(odd) 3/6
Appendix B Critical Reasoning: Section 10 Please download section 10 from the gm
atclub.com’s CR section Read through passages, but do not answer the questions, do
n’t even try. What you need to concentrate first is paraphrasing and pinpointing a
ssumptions – to check yourself, below you will find paraphrases and assumptions fo
r each question in chapter 10
Question # 3
Paraphrase Old study – eating chocolate increases chances of getting heart disease
; new study – chocolate does not increase heart disease chances; therefore, people
will buy more chocolate
Assumption People were not buying chocolate because they were afraid of heart di
sease
4
As climate of North America got hot and dry large mammals died while small ones
survived
This assumption is possible only after
 brining in an implied conclusion – change i
n climate killed the large animals. arge mammals were more
5
Higher profits will give higher bonuses, therefore, general economic recession y
ear will bring lower bonuses than year of profits
sensitive to heat than small ones The Industry will not bring profits in the yea
r of general economic recession
6
There’s less suitable area for gray wolves, however, bringing gray wolves back to
the places where they have been hunted out is immoral
People will keep hunting
8
Developed a safer equipment for Swedish market, brought it to US but doesn’t adver
tise this safety improvement
Safety improvement should be advertize because it will help him win over it’s comp
etitors Killing subsidy for the public bus system will not hurt the suburbs comm
unity
9
Few residents use public buses because there are enough automobiles, therefore,
public bus system should not be subsidized
14
A survey was conducted, according to is employees with high ratings ware satisfi
ed with the company’s system, therefore, the company’s best performing employees lik
e the system
Employees with high ratings are the company’s best performing employees
16
1970’s – fall in the average annual income of college graduates as compared to high
school graduates; 1980’s – the average annual income of college graduates increases
while the number of college graduates did not decrease
N/A
19
Industrialists are accused of intervention in war to make profit, however, feder
al expenses for intervention were larger than profits, therefore, the accusation
is wrongly motivated
Federal money (expenses and profits) and industrialists’ money come from the same
source
Appendix C Critical Reasoning: Section 11
Please download section 11 from the gmatclub.com’s CR section Read through passage
s, but do not answer the questions, don’t even try. What you need to concentrate f
irst is paraphrasing and pinpointing assumptions – to check yourself, below you wi
ll find paraphrases and assumptions for each question in chapter 11
Question # 1.
Paraphrase School board decided to reduce its staff by laying off the least effe
ctive teachers first
Assumption It is possible to determine the level of teachers’ effectiveness
2.
Applied scientific research is emphasized because it leads to technological adva
ncement; basic research should be paid attention to because it’s a base for applie
d research
Basic research does not lead to technological advancement
3.
Decision to invest in electronic system has cost advantage over nonelectronic sy
stem, therefore, it will give advantage over competitors
Competitors are not using the electronic system yet and won’t match it
5.
One museum sold 30K tickets, in a year 2 more museums opened and together the 3
of them sold 80K tickets, therefore, museums were worth the cost
The first old museum did not sell more tickets next year, as compared to previou
s year, and it is due to the two new museums that the number of tickets sold inc
reased
8.
Company needs to cut costs, so offer early retirement first, and then fire other
s to have the overall reduction of costs to 50 percent
Reducing costs by firing will not hurt company’s productivity: 50 percent of manag
ers left will be able to do the double work load Passengers will be willing to s
witch from air travel to trains
9.
Need to reduce airport congestion, so send passengers by rapid trains to several
cities while realizing that it is the major airport that is congested
12.
Received 2000 letters, most of them
People who wrote the letters
support him, therefore, most people in the country support him 13. Government wa
nted to regulate what can be shown in tobacco and alcohol advertisement, as a re
sult, these ads became more inventive and humorous
represent the opinion of majority
Implied conclusion: tobacco and alcohol advertisement should be banned. 1. Use o
f tobacco and alcohol can be influenced by advertisements 2. The funny and inven
tive commercials are more persuasive than the old ones.
15.

ead contamination dropped: federal regulation went into effect, but mainly beca
use there was a drop in the use of leaded gasoline
N/A – no conclusion
17.
Decrease in traditional child disease, and at the same time increase in rare inf
ection among children whereas few adults are affected
Children are more sensitive to rare infections than adults
18.
Plants come from less developed nations without compensation, but coal, oil and
ores are extracted for payment
The value and benefit from these plants are the same as from coal, oil, etc. The
y are comparable The number of visitors to see the exhibit will not override the
financial costs of keeping the exhibit
19.
Donates an exhibit which demands a storage space, a routine conservation, theref
ore, it adds to museum’s expenses and does not help it financially
20.
Oil prices are remaining low, therefore, natural gas prices will also be low
Oil and natural gas prices are interconnected
Appendix D General Strategies for Reading Comprehension, by Stephen Bolton 1. Tr
y to read the whole text of the passage once, if possible. Many people think you
should just skim the passage or read the first lines of every paragraph, and no
t to read the passage. We believe this is an error: if you misunderstand the mai
n idea of
the passage, you will certainly get at least some of the questions wrong. Give t
he passage one good read, taking no more than 3 minutes to read all of the text.
Do not read the passage more than once – that wastes too much time. If you have n
ot understood it completely, try to answer the questions anyway. Note: this poin
t of reading the whole passage is important for test-takers whose first language
is not English, provided that they can read the passage in 3 minutes or less.
2.
Make brief notes on the text on your scrap paper. As we will see below in greate
r detail, you should write down a couple of words on A) the Main Idea or Primary
Purpose, B) Organization/Structure of the passage, and C) the Tone or Attitude
of the author (if applicable). You just need a few words for each of these areas
, and altogether it should not take longer than 30 seconds to write down.
3.
Remember that the tone or attitude of the passage is usually respectful and mode
rate, never going to extremes of praise nor criticism. ETS obtains its Reading C
omprehension passages from real articles about real academics and professionals.
So the tone of the articles, even when there is criticism in the passage toward
an academic or her work, is always balanced and moderate. In the same vein, art
icles that deal with minorities or ethnic groups are almost always positive and
sympathetic.
4.

ook out for structural words that tell you the important ideas or transitions i
n a passage.

Continue the Idea Words Similarly Moreover Additionally In the same way ikewise
Conclusion Words Thus Therefore Hence So In summary In conclusion Contradiction
or Contrast Words Neverthless Nonetheless However But Although Though Even thoug
h Notwithstanding Yet Despite In spite of On the one hand…on the other hand While
Unlike
5.
Go back to the text of the passage for the answers. Many test-takers fail to ret
urn to the text of the passage to look for the correct answers. They rely solely
on their memories and understanding of the passage after having read or skimmed
it. Wrong. ETS is counting on that. Go back to the text to look for information
to answer the questions. Nine times out of ten, the answer lies within the pass
age.
Of the 6 most important types of questions for Reading Comprehension, we will fi
rst look at Main Idea/Primary Purpose Questions, and the strategies we can use t
o answer them.
Main Idea/Primary Purpose Questions
Many people believe there is no difference between the main or central idea of t
he passageand the primary purpose of the author of the passage. This is simply
not true. et s take a look at the subtle but important difference between them:
Main Idea The question might look something like this: "Which of the following
best states the central idea of the passage?" "Which of the following most accur
ately states the main idea of the passage?" "Which of the following is the princ
ipal topic of the passage?" "The main topic of the passage is...." Primary Purpo
se The question might look like this: "The primary purpose of this passage is to
..." "The primary purpose of the passage as a whole is to..." "The primary focus
of this passage is on which of the following?" "The main concern of the passage
is to..." "In the passage, the author is primarily interested
 in...." "The pass
age is chiefly concerned with..." Strategy: Main Idea: ook in the first and las
t paragraphs for the main idea. Any conclusion words like therefore, thus, so, h
ence, etc. that you see are most likely introducing the main idea. The correct a
nswer will say the same thing as it says in the text, but using different words.
The Main Idea is not always stated explicitly in the passage – in fact, more like
ly than not, it is not stated explicitly. Therefore, in order to answer this typ
e of question when it is more implicit:
1.
Re-read the first line of every passage, and the last line of the first and last
paragraphs. This should give you the general structure or outline of the argume
nt, with which you can answer the Main Idea question.
2.
After determining the general structure or content of the argument, eliminate an
swer choices that are too broad or too specific, i.e. answer choices that go bey
ond the content of the passage, or that deal with content only discussed in one
paragraph of the passage.
3.
Make brief notes – a couple of words- regarding the Main Idea on the text on your
scrap paper while you read.
Primary Purpose: What is the author trying to do? What is his intention? If he i
s evaluating a theory, then the answer could be something like "Discuss an inter
pretation". Note that the correct answer would deal with "an interpretation", be
cause the author is only dealing with one theory. If the Primary Purpose is to c
riticize 2 new books, then his intention or his primary purpose might be to "Cri
tique new studies". Again, as in Main Idea questions, re-read the first line of
every passage, and the last line of the first and last paragraphs. This should g
ive you the general structure or outline of the argument, with which you can ans
wer the Primary Purpose question. Note: A good main idea or primary purpose does
not go beyond the scope of the passage, nor does it limit itself to discussing
only one part of the passage. What is the primary purpose of this passage? discu
ss the importance of the television program Star Trek for the international spac
e A) program B) discuss important theoretical work concerned with faster-than-li
ght space travel. C) explore a dispute among theoretical physicists regarding th
e uses of space flight D) describe the possible uses of space-warping material E
) explain how a space-warping bubble would work in the real world Explanation Th
is is a Primary Purpose question, so we have to determine what the author is try
ing to do or say in this passage. So, let s read the first and last lines of the
passage in order to get an idea of the primary purpose. The first line says "Gr
eat news for Star Trek fans: warp drives that can propel starships around the Ga
laxy faster than the speed of light may be possible after all--with a little hel
p from Dr Who." The last line is a quote by a physicist that says "Of course, th
ere are still some basic questions--like how does one go about constructing this
Tardis space-time--but it puts the concept of space warps back on the agenda."
From both these sentences, we get the idea of space travel, faster than light tr
avel and space warps – maybe this is a discussion of faster than light space trave
l. Does that match what you have already read? Yes, basically this is a discussi
on of the theoretical state of play in the area of fasterthan-light space travel
. Do any of the 5 answer choices match that? Yes – B, even if the wording is somew
hat different from how we are wording it, the idea is almost exactly the same. B
is the answer. Another way of getting to the answer is through elimination of o
bviously incorrect answer choices. We can eliminate A because the author mention
s the popular science fiction program Star Trek merely to introduce the idea of
faster-than-light travel, and nothing more. C is a stronger possibility because
the second paragraph of the passage does discuss some disagreement among physici
sts about the possibility of creating a warp-drive, but in the same paragraph th
e theoretical dilemma seems resolved. Moreover, since the author only discusses
this in one paragraph, it cannot be the primary purpose of the entire passage. W
e can eliminate D because the author does not go into detail discussing the uses
of space-
warping material. And we can discard E because the author does not really go int
o how the space-warping bubble would work in the real world. Title Questions by
Stephen Bolton, 20th August, 1999 Title questions are very similar to Main Idea
questions, though are less common. Though some of the example paassage we use in
this tutorial and in the Practice Section are from the New Scientist, and there
fore have titles, the passages in the real GMAT will not have titles. The questi
on might look like this:
"Which of the following titles best summarizes the passage as a whole?"
Strategy: Treat this as a Main Idea question. A good title sums up the central i
dea of a passage. Therefore, in order to answer this type of question:
1.

ook in the first and last paragraphs for the main idea. Any conclusion words li
ke therefore, thus, so, hence, etc. that you see are most likely introducing the
Main Idea/Title. The correct answer will say the same thing as it says in the t
ext, but using different words.
2.
Re-read the first line of every passage, and the last line of the first and last
paragraphs. This should give you the general structure or outline of the argume
nt, with which you can answer the Title question.
3. 4.
Make brief notes – a couple of words- regarding the Title on the text on your scra
p paper while you read. After determining the general structure or content of th
e argument, eliminate answer choices that are too broad or too specific, i.e. an
swer choices that go beyond the content of the passage, or that deal with conten
t only discussed in one paragraph of the passage.
What would be an appropriate title for this passage? A) Constructing The Tardis

B) How To Make Space-Warping Material C) Bubbles In Space-Time D) Faster-Than- i
ght Travel: A Possibility? E) Debate On The Uses of Space Travel
Explanation This passage actually already has a title, "Warp Factor One". But we
have to look for another title possibility, one that would be most like the Mai
n Idea of the passage. We look at the first and last paragraphs, and since the M
ain Idea is that researchers now feel that faster-thanlight travel maybe more th
an mere fantasy, we can find the correct answer choice. Does any answer choice c
orrspond to this idea? Yes- answer D, which is the correct answer. We can also f
ind the correct answer through elimination. There is nowhere in the passage wher
e it discusses building Dr. Who s Tardis (pity!), so we can eliminate A. Nor doe
s it tell us how to make space-warping material. Eliminate B. While bubbles in s
pace-time are discussed at some length in one of the paragraphs, we cannot say t
his is the main concern of the passage, and thus should eliminate C. And nowhere
are the uses of space travel discussed, so discard E. Specific Detail or Target
questions are probably the most common types of questions, and the easiest to a
nswer. The question might look like this:
"According to the passage,...." "The passage states that ...."

Strategy The Specific Detail or Target
 that we are looking for could be a ine N
umber, or a Name or Date. Go to the ine Number or Name or Date, and then read s
everal lines above and below it. Find the answer choice that basically says the
same thing as in the passage, though usually with different words or word order.
According to the passage, Pfenning and Ford A) demonstrated conclusively the im
possibility of faster-than-light travel B) explored the possibility of bubbles t
hat warp space C) supported the work of Alcubierre D)work at of the Institute f
or Theoretical Physics at the Catholic University of euven E) suggested that a
warp drive was not physically possible ExplanationThis is a Specific Detail/Tar
get question, and therefore we look for the Name, ine Number, or Date that will
help us. In this case, the detail consists of the names Pfenning and Ford. We s
can the text, starting from the top of the passage, looking for the names Pfenni
ng and Ford. We find them in only place, at the beginning of the second paragrap
h. We read a couple of lines above the names, and keep reading
 until a few lines
after the names. It says "But in 1997 Michael Pfenning and arry Ford at Tufts
University in Medford, Massachusetts, apparently killed this ingenious idea by s
howing that it needed far more than the entire energy

content of the Universe to work (This Week, 26 uly 1997, p 6)". The line after
that
 says the research of another physicist then resurrected the possibility of
FT travel, negating the implications of the research of Pfenning and Ford. Now
we can answer the question. Do any ofthe answer choices match the information g
iven around the target area? Yes- E. et s also eliminate. If we re-read what th
e passage says about Pfenning and Ford, we can eliminate B, C, and D. None of th
em are supported by the information in the passage, so let s eliminate all of th
em without wasting too much time and with a minimum of fuss. A is tougher to eli
minate. From the sentence that mention Pfenning and Ford, it seems their work do
es rule out the possibility of a space-warp drive. But if we read the next line,
it says another researcher said it was indeed possible. So the Pfenning
 and For
d could not have "conclusively" demonstrated the impossibity of the FT drive. T
his is probably the most difficult type of Reading Comprehension problem. The qu
estion might look like this: "It can be inferred that the author makes which of
the following assumptions?" "Which is an assumption underlying the last sentence
of the passage?" "Which of the following, if true, would most strengthen the hy
pothesis mentioned in lines 17-19?" "With which of the following statements rega
rding chaos theory would the author be most likely to agree?"

Strategy: 1. First, treat this type of problem as a Specific Target question. o
ok for a target in the question, find it in the text, and then look above and be
low it. Often you do not have to infer very much, the answer remains within the
text. 2. If the answer must be inferred and is not stated explicitly within the
text, then choose the answer choice that can be inferred or assumed from the inf
ormation given. Again, you should not have to infer very much – only one or two lo
gical steps removed from the information in the passage. 3. Make sure that the a
nswer choice you decide on does not violate or contradict the Main Idea of the p
assage - if it does, the answer choice is probably wrong. It can be inferred tha
t a house with the propeties of the bubble mentioned in the passage A) would be
larger on the inside than on the outside B) could move faster than the speed of
light C) might be very energy efficient D) could move through time E) would even
tually fold in on itself and be destroyed Explanation First, let s try to deal w
ith this question as a Specific Target problem. Is there a target in the questio
n? Yes – the bubble. The bubble is first mentioned at the end of the second
paragraph, and then discussed at length throughout the third paragraph. Remember
, we have to look above and below that target area (as well as read the target a
rea again), so quickly go through the second, third, and first part of the fourt
h paragraph. When you are finished, look at the answer choices. Can any of them
be inferred from the information given in the target area? Well, we could elimin
ate C, D, and E for simply not being supported by the information given in the p
assage. B – maybe, but a house moving through time seems pretty silly. But in the
fourth paragraph the author talks about the Tardis, "which looked like a police
box but had a spacious interior". Big on the inside, small on the outside. Is th
at like our house? Yes- answer A. As well, we can choose A because it does not g
o against or contradict the Main Idea in this case, which if it had, would have
made it necessary to eliminate. So choose A. The question might look like this:
"The author s attitude
 towards Morgan s theory could best be described as one of
..." Strategy: ook for descriptive words, adjectives or adverbs, that could te
ll you the author s attitude. For example, the words unfortunately or flaw sugge
st a negative connotation, while strength or valuable emphasize the positive. Ma
ke brief notes – a couple of words- regarding the Tone of the text on your scrap p
aper while you read. Additionally, keep in mind that the author s attitude towar
d a theory, book, or ethnic group will almost always be respectful, even when so
mewhat critical. The author s attitude towards Miguel Alcibierre s theory could
best be described as one of A) Admiration B) mild scepticism C) unbridled scorn
D) Dismay E) complete objectivity Explanation Since this is a Tone/Attitude ques
tion, we must look in the passage for descriptive words that tell us what the au
thor thinks of Alcibierre and his theory. In the second paragraph the author cal
l s Alcibierre s theory "this ingenious idea". This is positive, and the only po
sitive answer choice is A. A is the correct answer. As well, if we could not fin
d the tone so easily, we could also eliminate C and D at the very least, for bei
ng too extreme. The question might look like this: "Which of the following best
describes the organization of the passage?" "Which of the following best describ
es the organization of the first paragraph of the passage?" "One function of the
third paragraph is to...." Strategy:
Re-read the first line of every passage, and the last line of the first and last
paragraphs. This should give you the general structure or outline of the argume
nt, with which you can answer the question. Remember to make brief notes about t
he structure of the text on your scrap paper. If you are looking for the organiz
ation of one paragraph, read the first and second sentence of the paragraph. Tha
t will give you a rough idea of what is the structure or organization of the par
agraph. Which of the following best describes the organization of the second par
agraph of the passage? A) Two investigations that support Alcubierre s theory ar
e introduced B) Possible objections to the uses of the warp drive are present, a
nd then refuted An objection to the practicality of the theory is raised, and th
en another work is cited to C) shore up the applicability of the original theory
A work of theoretical physics that supports Alcubierre s theory is raised, and
then D) another that refutes it is presented E) Alcubierre s theory is analyzed
by a panel of several eminent physicists Explanation
 Read the first sentence of
the paragraph: "But in 1997 Michael Pfenning and arry Ford at Tufts University
in Medford, Massachusetts, apparently killed this ingenious idea by showing that
it needed far more than the entire energy content of the Universe to work (This
Week, 26 uly 1997, p 6)". Then read the second sentence: "Now Chris Van Den  Br
oeck of the Institute for Theoretical Physics at the Catholic University of euv
en, Belgium, has resurrected Alcubierre s proposal". So if we out those two sent
ences together, and in different words, first the usefulness of Alcubierre s the
ory is questioned by two researchers, then the theory is validated by yet anothe
r researcher. Which of the answer choices is closest to this? C. None of the oth
er answer choices follow the organizational pattern of the paragraph – they revers
e it, or are completely dissimilar. C is the only possible answer. 1. Read the w
hole text of the passage once. 2. 3. 4. 5. Make brief notes about the text on yo
ur scrap paper. Remember that the tone or attitude of the passage is usually
 res
pectful and moderate, never going to extremes of praise nor criticism. ook out
for structural words that tell you the important ideas or transitions in a passa
ge. Go back to the text of the passage for the answers to specific questions.
Idiomatic Expressions, by Stephen Bolton Idiomatic expressions are phrases that
are common in standard written English. There is no magic formula or logical pro
cedure for determining if a particular combination of verbs,
prepositions, and nouns is correct – you simply have learned these expressions bef
orehand, or you have not. Here is an example of an idiomatic expression : I forb
id you to go. Why forbid to? Why not forbid from? Or forbid with? It is simply f
orbid to and not something else because centuries of custom and use have made it
that way. We must 1) accept that, and 2) learn that. 1. Watch for the prepositi
ons (to, the, of, at, for, on, in, about, etc.,) changing among the answer choic
es. This usually implies an Idiomatic problem, if not a Parallel Construction pr
oblem.
2.
Note that Idiomatic problems are often shorter than many of the other types of q
uestions. Not in all cases, but as a general rule, if the problem seems much sho
rter than other problems, it is probably Idiomatic.
3.
Check out the verb immediately before the changing preposition. Which verbprepos
ition combination sounds worst? Eliminate answer choices that sound just awful,
ex. He forbids me of going. The correct answer would be He forbids me to go.
4. Example
Which verb-preposition combination sounds best? Choose that as your answer.
The Duke of Argyle told us that we could count with him to be there on time. A w
ith him to be there B with him on being there C on him to be there D on him for
being there E to him to be there Explanation The correct answer is C. We know it
is an Idiomatic problem because a) the prepositions change among the answer cho
ices (with, with, on, on, and to), and 2) the problem is shorter than the averag
e Sentence Correction problem. We can eliminate count to for sounding unidiomati
c. So eliminate E. Count on seems better than count with, so eliminate A and B.
Between C and D, count on him to be there sounds better than count on him for be
ing there, so choose C as your correct answer.

Potrebbero piacerti anche